LSAT Explanation PT 45, S4, Q14: Insufficient rain can cause crops to

LSAT Question Stem

The argument's reasoning is most vulnerable to criticism on the grounds that the argument 

Logical Reasoning Question Type

This is a Flaw question. 

Correct Answer

The correct answer to this question is D. 

LSAT Question Complete Explanation

Let's first analyze the argument in the passage. The argument can be summarized as follows:

1. Insufficient rain can cause crops to falter and agricultural prices to rise (premise).

2. During a certain nation's recent crisis, faltering crops and rising agricultural prices led the government to take over food distribution (premise).

3. Therefore, the weather (specifically, insufficient rain) must have played an important role in causing the crisis (conclusion).

The structure of the argument is that it presents two premises and then draws a conclusion based on those premises. The argument is attempting to establish a causal relationship between insufficient rain and the nation's crisis.

An "Evaluate" question for this argument could be: "Were there any other factors that could have caused the faltering crops and rising agricultural prices during the nation's crisis?"

Now let's discuss the question type and what it's asking us to do. This is a Flaw question, which means we need to identify a flaw in the argument's reasoning.

a) This answer choice is incorrect because the argument doesn't merely conclude that insufficient rain occurred before the nation's crisis; it argues that insufficient rain caused the faltering crops and rising agricultural prices during the crisis.

b) This answer choice is off-topic because it focuses on the severity of the crisis and whether the government's takeover of food distribution was justified. The argument's flaw lies in its causal reasoning, not in the government's response to the crisis.

c) This answer choice is incorrect because the term "crisis" is used consistently throughout the passage, referring to the nation's crisis involving faltering crops and rising agricultural prices. There is no equivocation in the use of the term.

d) This is the correct answer choice. The argument's flaw is that it infers that because insufficient rain is a possible cause of faltering crops and rising agricultural prices, it must be the definite cause in this case. The argument does not provide sufficient evidence to establish this causal relationship.

e) This answer choice is incorrect because it describes an error of conditional reasoning, whereas the flaw in the argument is related to causal reasoning. The argument does not assume that any condition necessary for an increase in agricultural prices is also sufficient for such an increase.

Previous
Previous

LSAT Explanation PT 36, S3, Q3: Muscular strength is a limited resource,

Next
Next

LSAT Explanation PT 45, S1, Q14: Physician: In order to investigate diseases